0
$\begingroup$

I was taught that any orthogonal matrix must have eigenvalues of magnitude 1, i.e. $\lambda_i=\pm1,\forall i$. However, I am given the following matrix $$ M = \frac{1}{2}\begin{pmatrix} -1 & -1 & \sqrt{2} \\ 1 & 1 & \sqrt{2} \\ \sqrt{2} & -\sqrt{2} & 0 \end{pmatrix}. $$ We can see it is orthogonal by computing $MM^T$. We also find that $\det M=-1$, so the matrix corresponds to a reflection. Thus we know there is one eigenvalue $\lambda_1=-1$ and the transformation reflects vectors along the plane whose normal is the eigenvector corresponding to $\lambda_1$.

However, trying to find the other two eigenvalues through $\operatorname{tr}{M} = \lambda_1 + \lambda_2 + \lambda_3$ I get $$ \lambda_2 + \lambda_3 = 1, $$ which contradicts $\lambda_i=\pm1$. And indeed when I calculate the characteristic polynomial I get the equation $$ \lambda^3-\lambda+1=0, $$ which is not divisible by $(\lambda+1)$, which is yet another contradiction. But in my textbook it explicitly states that an orthogonal matrix has eigenvalues $\pm1$ and that a reflection has at least one eigenvalue equal to $-1$ (which is just common sense).

Can you help me understand where my reasoning is wrong? And how to solve the remaining eigenvalues and eigenvectors?

$\endgroup$
6
  • $\begingroup$ Tried to edit, but can't fix the formatting of the "\tr" function... $\endgroup$ Commented Feb 19 at 20:08
  • 7
    $\begingroup$ Eigenvalues do not have to be real, so $|\lambda| = 1$ does not imply $\lambda = \pm 1$. $\endgroup$ Commented Feb 19 at 20:09
  • 1
    $\begingroup$ Recheck your calculation of the characteristic polynomial. (Note that $\lambda=-1$ is not a root of $\lambda^3-\lambda+1=0$, so this can't be the right polynomial.) $\endgroup$ Commented Feb 19 at 20:11
  • $\begingroup$ characteristic polynomial is $x^3 + 1 .$ Indeed $M^3 = -I.$ The determinant is $-1$ but $M^2$ is not the identity, so $M$ does not create a reflection. A null vector for $M + I$ is $ v = (\sqrt 2, 0, -1)^T.$ So $v$ is an eigenvector. What is its eigenvalue? Which is to say, what is $Mv?$ $\endgroup$ Commented Feb 19 at 20:27
  • $\begingroup$ Consider a quarter turn rotation matrix in the $\mathbb{R}^2$. The eigenvalues will be $\pm i$ and $|\pm i|=1$ but the trace will be $0$. $\endgroup$ Commented Feb 20 at 1:13

0

You must log in to answer this question.

Start asking to get answers

Find the answer to your question by asking.

Ask question

Explore related questions

See similar questions with these tags.